LSAT and Law School Admissions Forum

Get expert LSAT preparation and law school admissions advice from PowerScore Test Preparation.

User avatar
 Dave Killoran
PowerScore Staff
  • PowerScore Staff
  • Posts: 5850
  • Joined: Mar 25, 2011
|
#44084
Complete Question Explanation
(The complete setup for this game can be found here: lsat/viewtopic.php?t=3405)

The correct answer choice is (B)

Answer choice (A) is incorrect because under the proposed scenario all four organisms would respond to F, a violation of the third rule. Answer choice (D) is incorrect because if W responds to G, then X responds to G, but since Y already responds to G, the condition in the answer choice cannot occur. Answer choice (C) is incorrect because every antibiotic that W responds to is also responded to by X, as specified in the fourth rule. Answer choice (E) is incorrect because every organism that responds to F also responds to G, as specified in the fifth rule. As a result, (B) is the correct answer choice.

Get the most out of your LSAT Prep Plus subscription.

Analyze and track your performance with our Testing and Analytics Package.